Difference between revisions of "2007 AMC 10B Problems/Problem 22"

(Solution)
(Solution)
Line 9: Line 9:
 
There are <math>2 \cdot 3 \cdot 1 = 6</math> ways for your number to show up once, <math>1 \cdot 1 = 1</math> way for your number to show up twice, and <math>3 \cdot 3 = 9</math> ways for your number to not show up at all. Think of this as a set of <math>16</math> numbers with six <math>1s,</math> one <math>2,</math> and nine <math>-1s.</math> The average of this set is the expected return to the player.
 
There are <math>2 \cdot 3 \cdot 1 = 6</math> ways for your number to show up once, <math>1 \cdot 1 = 1</math> way for your number to show up twice, and <math>3 \cdot 3 = 9</math> ways for your number to not show up at all. Think of this as a set of <math>16</math> numbers with six <math>1s,</math> one <math>2,</math> and nine <math>-1s.</math> The average of this set is the expected return to the player.
 
<cmath>\frac{6(1)+1(2)-9(1)}{16} = \frac{6+2-9}{16} = \boxed{\mathrm{(B) \ } -\frac{1}{16}}</cmath>
 
<cmath>\frac{6(1)+1(2)-9(1)}{16} = \frac{6+2-9}{16} = \boxed{\mathrm{(B) \ } -\frac{1}{16}}</cmath>
 
  
 
==Solution 2==
 
==Solution 2==

Revision as of 18:26, 2 April 2021

Problem 22

A player chooses one of the numbers $1$ through $4$. After the choice has been made, two regular four-sided (tetrahedral) dice are rolled, with the sides of the dice numbered $1$ through $4.$ If the number chosen appears on the bottom of exactly one die after it has been rolled, then the player wins $1$ dollar. If the number chosen appears on the bottom of both of the dice, then the player wins $2$ dollars. If the number chosen does not appear on the bottom of either of the dice, the player loses $1$ dollar. What is the expected return to the player, in dollars, for one roll of the dice?

$\textbf{(A) } -\frac{1}{8} \qquad\textbf{(B) } -\frac{1}{16} \qquad\textbf{(C) } 0 \qquad\textbf{(D) } \frac{1}{16} \qquad\textbf{(E) } \frac{1}{8}$

Solution

There are $2 \cdot 3 \cdot 1 = 6$ ways for your number to show up once, $1 \cdot 1 = 1$ way for your number to show up twice, and $3 \cdot 3 = 9$ ways for your number to not show up at all. Think of this as a set of $16$ numbers with six $1s,$ one $2,$ and nine $-1s.$ The average of this set is the expected return to the player. \[\frac{6(1)+1(2)-9(1)}{16} = \frac{6+2-9}{16} = \boxed{\mathrm{(B) \ } -\frac{1}{16}}\]

Solution 2

We approach this through casework. We have a $\frac{1}{4} \cdot \frac{3}{4} \cdot 2$ chance of earning $1$ dollar. We have a $\frac{1}{4} \cdot \frac{1}{4}$ chance of earning 2 dollars, and a $\frac{3}{4} \cdot \frac{3}{4}$ chance of losing 1 dollar. Thus, our final answer is $\frac{3}{8} \cdot 1 + \frac{1}{16} \cdot 2 + \frac{9}{16} \cdot -1 = \boxed{\mathrm{(B) \ } -\frac{1}{16}}$ -SuperJJ

See Also

2007 AMC 10B (ProblemsAnswer KeyResources)
Preceded by
Problem 21
Followed by
Problem 23
1 2 3 4 5 6 7 8 9 10 11 12 13 14 15 16 17 18 19 20 21 22 23 24 25
All AMC 10 Problems and Solutions

The problems on this page are copyrighted by the Mathematical Association of America's American Mathematics Competitions. AMC logo.png